www.matheraum.de
Das Matheforum.
Das Matheforum des MatheRaum.

Für Schüler, Studenten, Lehrer, Mathematik-Interessierte.
Hallo Gast!einloggen | registrieren ]
Startseite · Forum · Wissen · Kurse · Mitglieder · Team · Impressum
Forenbaum
^ Forenbaum
Status Mathe
  Status Schulmathe
    Status Primarstufe
    Status Mathe Klassen 5-7
    Status Mathe Klassen 8-10
    Status Oberstufenmathe
    Status Mathe-Wettbewerbe
    Status Sonstiges
  Status Hochschulmathe
    Status Uni-Analysis
    Status Uni-Lin. Algebra
    Status Algebra+Zahlentheo.
    Status Diskrete Mathematik
    Status Fachdidaktik
    Status Finanz+Versicherung
    Status Logik+Mengenlehre
    Status Numerik
    Status Uni-Stochastik
    Status Topologie+Geometrie
    Status Uni-Sonstiges
  Status Mathe-Vorkurse
    Status Organisatorisches
    Status Schule
    Status Universität
  Status Mathe-Software
    Status Derive
    Status DynaGeo
    Status FunkyPlot
    Status GeoGebra
    Status LaTeX
    Status Maple
    Status MathCad
    Status Mathematica
    Status Matlab
    Status Maxima
    Status MuPad
    Status Taschenrechner

Gezeigt werden alle Foren bis zur Tiefe 2

Navigation
 Startseite...
 Neuerdings beta neu
 Forum...
 vorwissen...
 vorkurse...
 Werkzeuge...
 Nachhilfevermittlung beta...
 Online-Spiele beta
 Suchen
 Verein...
 Impressum
Das Projekt
Server und Internetanbindung werden durch Spenden finanziert.
Organisiert wird das Projekt von unserem Koordinatorenteam.
Hunderte Mitglieder helfen ehrenamtlich in unseren moderierten Foren.
Anbieter der Seite ist der gemeinnützige Verein "Vorhilfe.de e.V.".
Partnerseiten
Mathe-Seiten:Weitere Fächer:

Open Source FunktionenplotterFunkyPlot: Kostenloser und quelloffener Funktionenplotter für Linux und andere Betriebssysteme
StartseiteMatheForenAlgebraExistenz des Minimalpolynoms
Foren für weitere Schulfächer findest Du auf www.vorhilfe.de z.B. Informatik • Physik • Technik • Biologie • Chemie
Forum "Algebra" - Existenz des Minimalpolynoms
Existenz des Minimalpolynoms < Algebra < Algebra+Zahlentheo. < Hochschule < Mathe < Vorhilfe
Ansicht: [ geschachtelt ] | ^ Forum "Algebra"  | ^^ Alle Foren  | ^ Forenbaum  | Materialien

Existenz des Minimalpolynoms: Frage (beantwortet)
Status: (Frage) beantwortet Status 
Datum: 21:47 Fr 27.03.2020
Autor: teskiro

Guten Abend, ich bin es wieder. Beim Bearbeiten meiner Aufgaben komme ich nicht drum herum, mir davor den Beweis von zwei Sätzen anzuschauen, weil im Beweis des zweiten Satzes kommt ein Algorithmus vor, den ich für die eigentliche Aufgabe brauche.

Aber den Beweis des zweiten Satzes verstehe ich nicht, wenn ich nicht den ersten Satz verstehe.

Daher habe ich ein paar Fragen zum Beweis des ersten Satzes.
Ich tippe den Satz + Beweis mal ab und erkläre, wie ich den Beweis verstanden habe.

Ich meine, relativ weit gekommen zu sein, aber ein paar Sachen verwirren mich noch.




Satz:


Es sei $L/K$ eine Körpererweiterung und [mm] $\alpha \in [/mm] L$ sei algebraisch über $K$.

a) Es gibt ein eindeutig bestimmtes normiertes Polynom $ 0 [mm] \neq \mu_{\alpha} \in K[\; t\;]$ [/mm] kleinsten Grades, das [mm] $\alpha$ [/mm] als Nullstelle hat, das Minimalpolynom von [mm] $\alpha$ [/mm] über $K$.

b)  Das Minimalpolynom [mm] $\mu_{\alpha}$ [/mm] erzeugt den Kern des Einsetzhomomorphismus [mm] $\Phi_{\alpha}$ [/mm]





Beweis:



Der Kern von [mm] $\Phi_{\alpha}$ [/mm] wird als Ideal im Hauptidealring [mm] $K[\; t\;]$ [/mm] von einem Polynom [mm] $\mu_{\alpha}$ [/mm] erzeugt. Indem wir durch den Leitkoeffizienten teilen, können wir ohne Einschränkungen annehmen, dass [mm] $\mu_{\alpha}$ [/mm] normiert ist.


Ist $0 [mm] \neq [/mm] f [mm] \in K[\; t\;]$ [/mm] irgendein Polynom, dass [mm] $\alpha$ [/mm] als Nullstelle hat, so gilt


$f [mm] \in Ker(\Phi_{\alpha}) [/mm] = [mm] \langle \mu_{\alpha}\rangle$. [/mm]


Mithin gibt es ein $g [mm] \in K[\; t\;]$ [/mm] mit $f = g [mm] \cdot \mu_{\alpha}$, [/mm]   (17)


und aus der Gradformel folgt $deg(f) = deg(g) + [mm] deg(\mu_{\alpha}) \ge deg(\mu_{\alpha})$, [/mm]    (18)

so dass [mm] $\mu_{\alpha}$ [/mm] in der Tat ein Nicht- Null- Polynom kleinsten Grades ist, dass [mm] $\alpha$ [/mm] als Nullstelle besitzt.

Um die Eindeutigkeit von [mm] $\mu_{\alpha}$ [/mm] zu sehen, können wir annehmen, dass $f$ ein zweites normiertes Polynom kleinsten Grades mit [mm] $\alpha$ [/mm] als Nullstelle ist.


Aus $(18)$ sehen wir dann, dass der Faktor $g$ in $(17)$ konstant sein muss, und da $f$ und [mm] $\mu_{\alpha}$ [/mm] beide normiert sind, muss in der Tat $g = 1$ und damit $f = [mm] \mu_{\alpha}$ [/mm] gelten.

Damit sind Teil a) und Teil b) gezeigt.











Ich werde mal aufschreiben,  wie ich den Beweis verstanden habe und werde meine Fragen zu einem Abschnitt in eckigen Klammern schreiben.







Wir haben die Körpererweiterung $L/K$, d.h. $K$ ist ein Teilkörper von $L$.

Wir betrachten den Einsetzhomomorphismus  [mm] $\Phi_{\alpha}: K[\;t\;] \rightarrow [/mm] L, f [mm] \mapsto [/mm] f(b)$

Diese Abbildung ist ein Ringhomomorphismus, daher ist [mm] $Ker(\Phi_{\alpha})$ [/mm] ein Ideal von [mm] $K[\;t\;]$. [/mm]


Polynomringe über Körper sind Hauptidealringe und in einem Hauptidealring ist jedes Ideal ein Hauptideal.

Also ist [mm] $Ker(\Phi_{\alpha})$ [/mm] ein Hauptideal von [mm] $K[\;t\;]$. [/mm]


Da  [mm] $Ker(\Phi_{\alpha})$ [/mm] ein Hauptideal von [mm] $K[\;t\;]$ [/mm] ist, wird  [mm] $Ker(\Phi_{\alpha})$ [/mm] von einem Polynom [mm] $\mu_{\alpha} \in K[\; [/mm] t [mm] \;]$ [/mm] erzeugt.

Es gilt also  [mm] $Ker(\Phi_{\alpha}) [/mm] = [mm] \langle \mu_{\alpha} \rangle [/mm] = [mm] \{ g \cdot \mu_{\alpha}\; \vert \; g \in K[\; t\;] \}$ [/mm]



[Frage 1: Eine Abbildung zwischen zwei Körpern, die ein Körperhomomorphismus ist, ist injektiv. Das heißt, der Kern dieser Abbildung ist trivial.

Die Abbildung [mm] $\Phi_{\alpha}$ [/mm] ist ein Körperhomomorphismus. Muss also nicht [mm] $Ker(\Phi_{\alpha})$ [/mm] nur aus dem Nullpolynom von [mm] $K[\; [/mm] t [mm] \; [/mm] ]$ bestehen ? ]





[ Frage 2: Angenommen, der Kern [mm] $Ker(\Phi_{\alpha})$ [/mm]  ist nicht trivial.

Was meint man mit "Indem wir durch den Leitkoeffizienten teilen, können wir ohne Einschränkungen annehmen, dass [mm] $\mu_{\alpha}$ [/mm] normiert ist" ?

Normieren wir das Polynom einfach ? Was ist, wenn  [mm] $\mu_{\alpha} [/mm] = 0$ ? Diesen Fall haben wir nämlich noch nicht ausgeschlossen]



Sei $0 [mm] \neq [/mm] f [mm] \in K[\; [/mm] t [mm] \; [/mm] ]$ ein Polynom mit [mm] $f(\alpha) [/mm] = 0$.

Dann ist $f [mm] \in Ker(\Phi_{\alpha}) [/mm] = [mm] \langle \mu_{\alpha} \rangle [/mm] =  [mm] \{ g \cdot \mu_{\alpha}\; \vert \; g \in K[\; t\;] \}$ [/mm]

Mithin gibt es ein $g [mm] \in K[\; t\;]$ [/mm] mit $f = g [mm] \cdot \mu_{\alpha}$. [/mm]


[Frage 3: Müsste nicht $f [mm] \in K[\; [/mm] t [mm] \; [/mm] ] [mm] \setminus [/mm] K$ sein ? Weil konstante Polynome, außer das Nullpolynom, haben keine Nullstelle.]

[Frage 4: Wir nehmen ja an, dass [mm] $\mu_{\alpha}$ [/mm] nicht konstant ist. Dann darf aber $g$ durch aus konstant sein, oder ?

Aber dann müsste $g [mm] \in K[\; [/mm] t [mm] \; [/mm] ] [mm] \setminus \{ 0 \}$ [/mm] sein, denn sonst könnte $f = 0$ sein und diesen Fall haben wir oben ausgeschlossen.]




Wir haben also die Polynome $f = g [mm] \cdot \mu_{\alpha}$ [/mm] und [mm] $\mu_{\alpha}$, [/mm] die beide [mm] $\alpha$ [/mm] als Nullstelle haben. Fragt sich nur, welches Polynom den kleineren Grad hat.


Da $K$ ein nullteilerfreier Ring ist, gilt die Gradformel

$deg(f) = [mm] \underbrace{deg(g)}_{\ge 1} [/mm] + [mm] \underbrace{ \deg(\mu_{\alpha})}_{\ge 1} \ge \deg(\mu_{\alpha}) [/mm] $


[Frage 5: Diese Frage ist vielleicht banal, aber warum kann z.B. das Polynom $g$ das Element [mm] $\alpha$ [/mm] nicht als Nullstelle haben ? Es kann ja sein, dass $g$ ein Vielfaches von [mm] $\mu_{\alpha}$ [/mm] ist und damit [mm] $\alpha$ [/mm] als Nullstelle hat. ]



Das ist ein bisschen viel Text, aber ich hoffe, das stört nicht.


Viele Grüße, Teskiro


        
Bezug
Existenz des Minimalpolynoms: Antwort
Status: (Antwort) fertig Status 
Datum: 16:36 Sa 28.03.2020
Autor: statler

Hallo!


> Satz:
>
> Es sei [mm]L/K[/mm] eine Körpererweiterung und [mm]\alpha \in L[/mm] sei
> algebraisch über [mm]K[/mm].
>  
> a) Es gibt ein eindeutig bestimmtes normiertes Polynom [mm]0 \neq \mu_{\alpha} \in K[\; t\;][/mm]
> kleinsten Grades, das [mm]\alpha[/mm] als Nullstelle hat, das
> Minimalpolynom von [mm]\alpha[/mm] über [mm]K[/mm].
>  
> b)  Das Minimalpolynom [mm]\mu_{\alpha}[/mm] erzeugt den Kern des
> Einsetzhomomorphismus [mm]\Phi_{\alpha}[/mm]
>
> Beweis:
>  
> Der Kern von [mm]\Phi_{\alpha}[/mm] wird als Ideal im Hauptidealring
> [mm]K[\; t\;][/mm] von einem Polynom [mm]\mu_{\alpha}[/mm] erzeugt. Indem wir
> durch den Leitkoeffizienten teilen, können wir ohne
> Einschränkungen annehmen, dass [mm]\mu_{\alpha}[/mm] normiert ist.
>  
>
> Ist [mm]0 \neq f \in K[\; t\;][/mm] irgendein Polynom, dass [mm]\alpha[/mm]
> als Nullstelle hat, so gilt
>  
>
> [mm]f \in Ker(\Phi_{\alpha}) = \langle \mu_{\alpha}\rangle[/mm].
>  
>
> Mithin gibt es ein [mm]g \in K[\; t\;][/mm] mit [mm]f = g \cdot \mu_{\alpha}[/mm],
>   (17)
>  
>
> und aus der Gradformel folgt [mm]deg(f) = deg(g) + deg(\mu_{\alpha}) \ge deg(\mu_{\alpha})[/mm],
>    (18)
>  
> so dass [mm]\mu_{\alpha}[/mm] in der Tat ein Nicht- Null- Polynom
> kleinsten Grades ist, dass [mm]\alpha[/mm] als Nullstelle besitzt.
>  
> Um die Eindeutigkeit von [mm]\mu_{\alpha}[/mm] zu sehen, können wir
> annehmen, dass [mm]f[/mm] ein zweites normiertes Polynom kleinsten
> Grades mit [mm]\alpha[/mm] als Nullstelle ist.
>  
>
> Aus [mm](18)[/mm] sehen wir dann, dass der Faktor [mm]g[/mm] in [mm](17)[/mm] konstant
> sein muss, und da [mm]f[/mm] und [mm]\mu_{\alpha}[/mm] beide normiert sind,
> muss in der Tat [mm]g = 1[/mm] und damit [mm]f = \mu_{\alpha}[/mm] gelten.
>  
> Damit sind Teil a) und Teil b) gezeigt.
>  
>
>
>
>
> Ich werde mal aufschreiben,  wie ich den Beweis verstanden
> habe und werde meine Fragen zu einem Abschnitt in eckigen
> Klammern schreiben.
>  

> Wir haben die Körpererweiterung [mm]L/K[/mm], d.h. [mm]K[/mm] ist ein
> Teilkörper von [mm]L[/mm].
>  
> Wir betrachten den Einsetzhomomorphismus  [mm]\Phi_{\alpha}: K[\;t\;] \rightarrow L, f \mapsto f(b)[/mm]

$f [mm] \mapsto f(\alpha)$ [/mm]

>  
> Diese Abbildung ist ein Ringhomomorphismus, daher ist
> [mm]Ker(\Phi_{\alpha})[/mm] ein Ideal von [mm]K[\;t\;][/mm].
>  
>
> Polynomringe über Körper sind Hauptidealringe und in
> einem Hauptidealring ist jedes Ideal ein Hauptideal.
>
> Also ist [mm]Ker(\Phi_{\alpha})[/mm] ein Hauptideal von [mm]K[\;t\;][/mm].
>  
>
> Da  [mm]Ker(\Phi_{\alpha})[/mm] ein Hauptideal von [mm]K[\;t\;][/mm] ist,
> wird  [mm]Ker(\Phi_{\alpha})[/mm] von einem Polynom [mm]\mu_{\alpha} \in K[\; t \;][/mm]
> erzeugt.
>  
> Es gilt also  [mm]Ker(\Phi_{\alpha}) = \langle \mu_{\alpha} \rangle = \{ g \cdot \mu_{\alpha}\; \vert \; g \in K[\; t\;] \}[/mm]
>
>
>
> [Frage 1: Eine Abbildung zwischen zwei Körpern, die ein
> Körperhomomorphismus ist, ist injektiv. Das heißt, der
> Kern dieser Abbildung ist trivial.
>  
> Die Abbildung [mm]\Phi_{\alpha}[/mm] ist ein Körperhomomorphismus.
> Muss also nicht [mm]Ker(\Phi_{\alpha})[/mm] nur aus dem Nullpolynom
> von [mm]K[\; t \; ][/mm] bestehen ? ]

[mm] $\Phi_{\alpha}$ [/mm] ist kein Körperhomomorphismus, weil K[t] kein Körper ist (t hat kein Inverses.)

>
> [ Frage 2: Angenommen, der Kern [mm]Ker(\Phi_{\alpha})[/mm]  ist
> nicht trivial.
>  
> Was meint man mit "Indem wir durch den Leitkoeffizienten
> teilen, können wir ohne Einschränkungen annehmen, dass
> [mm]\mu_{\alpha}[/mm] normiert ist" ?

In K[t] sind die Elemente von K* Einheiten, das normierte Polynom erzeugt dasselbe Ideal.

>  
> Normieren wir das Polynom einfach ? Was ist, wenn  
> [mm]\mu_{\alpha} = 0[/mm] ? Diesen Fall haben wir nämlich noch
> nicht ausgeschlossen]
>  

[mm] $\mu_{\alpha} [/mm] = 0$ kann nicht passieren, da [mm] \alpha [/mm] algebraisch ist.
  

> Sei [mm]0 \neq f \in K[\; t \; ][/mm] ein Polynom mit [mm]f(\alpha) = 0[/mm].
>  
> Dann ist [mm]f \in Ker(\Phi_{\alpha}) = \langle \mu_{\alpha} \rangle = \{ g \cdot \mu_{\alpha}\; \vert \; g \in K[\; t\;] \}[/mm]
>  
> Mithin gibt es ein [mm]g \in K[\; t\;][/mm] mit [mm]f = g \cdot \mu_{\alpha}[/mm].
>  
>
> [Frage 3: Müsste nicht [mm]f \in K[\; t \; ] \setminus K[/mm] sein
> ? Weil konstante Polynome, außer das Nullpolynom, haben
> keine Nullstelle.]

Aber was bringt das?

>  
> [Frage 4: Wir nehmen ja an, dass [mm]\mu_{\alpha}[/mm] nicht
> konstant ist. Dann darf aber [mm]g[/mm] durch aus konstant sein,
> oder ?
>  
> Aber dann müsste [mm]g \in K[\; t \; ] \setminus \{ 0 \}[/mm] sein,
> denn sonst könnte [mm]f = 0[/mm] sein und diesen Fall haben wir
> oben ausgeschlossen.]
>  

Und was bringt das?

>
> Wir haben also die Polynome [mm]f = g \cdot \mu_{\alpha}[/mm] und
> [mm]\mu_{\alpha}[/mm], die beide [mm]\alpha[/mm] als Nullstelle haben. Fragt
> sich nur, welches Polynom den kleineren Grad hat.
>  
>
> Da [mm]K[/mm] ein nullteilerfreier Ring ist, gilt die Gradformel
>  
> [mm]deg(f) = \underbrace{deg(g)}_{\ge 1} + \underbrace{ \deg(\mu_{\alpha})}_{\ge 1} \ge \deg(\mu_{\alpha})[/mm]

Der Grad von g könnte auch 0 sein, das macht aber nichts.

>  
> [Frage 5: Diese Frage ist vielleicht banal, aber warum kann
> z.B. das Polynom [mm]g[/mm] das Element [mm]\alpha[/mm] nicht als Nullstelle
> haben ? Es kann ja sein, dass [mm]g[/mm] ein Vielfaches von
> [mm]\mu_{\alpha}[/mm] ist und damit [mm]\alpha[/mm] als Nullstelle hat. ]
>  

Auch das macht nichts.

>

Gruß aus HH
Dieter

Bezug
                
Bezug
Existenz des Minimalpolynoms: Frage (beantwortet)
Status: (Frage) beantwortet Status 
Datum: 20:59 So 29.03.2020
Autor: teskiro

Hallo! :)


> > Was meint man mit "Indem wir durch den Leitkoeffizienten
> > teilen, können wir ohne Einschränkungen annehmen, dass
> > [mm]\mu_{\alpha}[/mm] normiert ist" ?
>  
> In K[t] sind die Elemente von K* Einheiten, das normierte Polynom erzeugt dasselbe Ideal.


Es gilt also [mm] $\langle [/mm] k [mm] \cdot \mu_{\alpa} \rangle [/mm] = [mm] \langle \mu_{\alpa} \rangle$ [/mm] für $k [mm] \in [/mm] K$ und [mm] $\langle [/mm] q [mm] \cdot \mu_{\alpa} \rangle \neq \langle \mu_{\alpa} \rangle$ [/mm] für $q [mm] \in K[\; [/mm] t [mm] \; [/mm] ] [mm] \setminus [/mm] K$ ?



> > [Frage 3: Müsste nicht [mm]f \in K[\; t \; ] \setminus K[/mm] sein
> > ? Weil konstante Polynome, außer das Nullpolynom, haben
> > keine Nullstelle.]
>  
> Aber was bringt das?

Na ja, in der Mathematik ist man sonst immer so genau und schreibt hin, aus welcher Menge genau die Elemente kommen sollen.

Daher wundere ich mich ein wenig. Weil wenn $0 [mm] \neq [/mm] f [mm] \in K[\;t\;] [/mm] $ ist, werden Polynome zugelassen, die gar keine Nullstelle haben können, nämlich die konstanten Polynome ungleich Null.


> > Aber dann müsste [mm]g \in K[\; t \; ] \setminus \{ 0 \}[/mm] sein,
> > denn sonst könnte [mm]f = 0[/mm] sein und diesen Fall haben wir
> > oben ausgeschlossen.]
>  >  
> Und was bringt das?

Habe ich aus dem selben Grund geschrieben wie oben.

Wenn $g [mm] \in K[\; [/mm] t [mm] \; [/mm] ] $, dann wird z.B. $g = 0$ zugelassen und somit hätten wir  $f = g [mm] \cdot mu_{\alpha} [/mm] = 0 [mm] \cdot mu_{\alpha} [/mm] = 0$.

Und da macht es, meiner Meinung nach, keinen Sinn $0 [mm] \neq [/mm] f [mm] \in K[\; [/mm] t [mm] \; [/mm] ] $ zu schreiben, wenn sehr wohl $f = 0$ gelten kann.

>  >

> > Wir haben also die Polynome [mm]f = g \cdot \mu_{\alpha}[/mm] und
> > [mm]\mu_{\alpha}[/mm], die beide [mm]\alpha[/mm] als Nullstelle haben. Fragt
> > sich nur, welches Polynom den kleineren Grad hat.
>  >  
> >
> > Da [mm]K[/mm] ein nullteilerfreier Ring ist, gilt die Gradformel
>  >  
> > [mm]deg(f) = \underbrace{deg(g)}_{\ge 1} + \underbrace{ \deg(\mu_{\alpha})}_{\ge 1} \ge \deg(\mu_{\alpha})[/mm]
>  
> Der Grad von g könnte auch 0 sein, das macht aber nichts.
>  >  
> > [Frage 5: Diese Frage ist vielleicht banal, aber warum kann
> > z.B. das Polynom [mm]g[/mm] das Element [mm]\alpha[/mm] nicht als Nullstelle
> > haben ? Es kann ja sein, dass [mm]g[/mm] ein Vielfaches von
> > [mm]\mu_{\alpha}[/mm] ist und damit [mm]\alpha[/mm] als Nullstelle hat. ]
>  >  
> Auch das macht nichts.


Ist das unbedingt notwendig, die Gradformel in diesem Beweis anzuwenden ?

Dass [mm] $\mu_{\alpha}$ [/mm] kleinsten Grad ist, folgt schon daraus, dass die Gleichung [mm] $Ker(\Phi_{\alpha}) [/mm] = [mm] \langle \mu_{\alpha}\rangle [/mm] $ gilt.

Das heiß, dass  [mm] $\mu_{\alpha}$ [/mm] ein Teiler aller Polynome aus [mm] $K[\; [/mm] t [mm] \; [/mm] ]$ ist, die [mm] $\alpha \in [/mm] L$ als Nullstelle haben.

Und da ist es klar, dass [mm] $\mu_{\alpha}$ [/mm] kleinsten Grad haben muss.




Viele Grüße,

Tim

Bezug
                        
Bezug
Existenz des Minimalpolynoms: Antwort
Status: (Antwort) fertig Status 
Datum: 12:43 Mi 01.04.2020
Autor: statler

Mahlzeit!

> > > Was meint man mit "Indem wir durch den Leitkoeffizienten
> > > teilen, können wir ohne Einschränkungen annehmen, dass
> > > [mm]\mu_{\alpha}[/mm] normiert ist" ?
>  >  
> > In K[t] sind die Elemente von K* Einheiten, das normierte Polynom erzeugt dasselbe Ideal.
>  
>
> Es gilt also [mm]\langle k \cdot \mu_{\alpa} \rangle = \langle \mu_{\alpa} \rangle[/mm] für [mm]k \in K[/mm] und [mm]\langle q \cdot \mu_{\alpa} \rangle \neq \langle \mu_{\alpa} \rangle[/mm] für [mm]q \in K[\; t \; ] \setminus K[/mm] ?

Du mußt die Ausnahmefälle k = 0 und [mm] $\mu [/mm] = [mm] \langle [/mm] 0 [mm] \rangle$ [/mm] gesondert behandeln, in der obigen allgemeinen Form wäre die Aussage falsch.

> > > [Frage 3: Müsste nicht [mm]f \in K[\; t \; ] \setminus K[/mm] sein
> > > ? Weil konstante Polynome, außer das Nullpolynom, haben
> > > keine Nullstelle.]
>  >  
> > Aber was bringt das?
>  
> Na ja, in der Mathematik ist man sonst immer so genau und schreibt hin, aus welcher Menge genau die Elemente kommen sollen.
>  
> Daher wundere ich mich ein wenig. Weil wenn [mm]0 \neq f \in K[\;t\;][/mm] ist, werden Polynome zugelassen, die gar keine Nullstelle haben können, nämlich die konstanten Polynome ungleich Null.

Wenn $A [mm] \subseteq [/mm] B$ und ich etwas über die Elemente aus $B$ beweisen kann, dann habe ich das doch für Elemente aus $A$ auch bewiesen.
Man beweist Aussagen auch gerne so allgemein wie möglich, besonders dann, wenn das ohne Mehraufwand geht.

> > > Aber dann müsste [mm]g \in K[\; t \; ] \setminus \{ 0 \}[/mm] sein,
> > > denn sonst könnte [mm]f = 0[/mm] sein und diesen Fall haben wir
> > > oben ausgeschlossen.]
>  >  >  
> > Und was bringt das?
>  
> Habe ich aus dem selben Grund geschrieben wie oben.
>  
> Wenn [mm]g \in K[\; t \; ] [/mm], dann wird z.B. [mm]g = 0[/mm] zugelassen und somit hätten wir  [mm]f = g \cdot mu_{\alpha} = 0 \cdot mu_{\alpha} = 0[/mm].
>  
> Und da macht es, meiner Meinung nach, keinen Sinn [mm]0 \neq f \in K[\; t \; ][/mm] zu schreiben, wenn sehr wohl [mm]f = 0[/mm] gelten kann.

Du machst dir nach meinem Eindruck die Probleme selbst. Du kannst doch zunächst Dinge zulassen, von denen sich im Verlauf der Rechnung herausstellt, daß sie nicht eintreten können.

> > > Wir haben also die Polynome [mm]f = g \cdot \mu_{\alpha}[/mm] und
> > > [mm]\mu_{\alpha}[/mm], die beide [mm]\alpha[/mm] als Nullstelle haben. Fragt
> > > sich nur, welches Polynom den kleineren Grad hat.
>  >  >  
> > >
> > > Da [mm]K[/mm] ein nullteilerfreier Ring ist, gilt die Gradformel
>  >  >  
> > > [mm]deg(f) = \underbrace{deg(g)}_{\ge 1} + \underbrace{ \deg(\mu_{\alpha})}_{\ge 1} \ge \deg(\mu_{\alpha})[/mm]
>  >  
> > Der Grad von g könnte auch 0 sein, das macht aber nichts.
>  >  >  
> > > [Frage 5: Diese Frage ist vielleicht banal, aber warum kann
> > > z.B. das Polynom [mm]g[/mm] das Element [mm]\alpha[/mm] nicht als Nullstelle
> > > haben ? Es kann ja sein, dass [mm]g[/mm] ein Vielfaches von
> > > [mm]\mu_{\alpha}[/mm] ist und damit [mm]\alpha[/mm] als Nullstelle hat. ]
>  >  >  
> > Auch das macht nichts.
>  
>
> Ist das unbedingt notwendig, die Gradformel in diesem Beweis anzuwenden ?
>  
> Dass [mm]\mu_{\alpha}[/mm] kleinsten Grad ist, folgt schon daraus, dass die Gleichung [mm]Ker(\Phi_{\alpha}) = \langle \mu_{\alpha}\rangle[/mm] gilt.
>  
> Das heiß, dass  [mm]\mu_{\alpha}[/mm] ein Teiler aller Polynome aus [mm]K[\; t \; ][/mm] ist, die [mm]\alpha \in L[/mm] als Nullstelle haben.
>  
> Und da ist es klar, dass [mm]\mu_{\alpha}[/mm] kleinsten Grad haben muss.
>  

Und warum ist das klar?

Gruß Dieter

Bezug
                                
Bezug
Existenz des Minimalpolynoms: Frage (beantwortet)
Status: (Frage) beantwortet Status 
Datum: 15:01 Do 02.04.2020
Autor: teskiro

Guten Nachmittag :)

Ich bin gestern meine Überlegungen durchgegangen und du hast Recht, ich mache mir selber die Probleme.

Ich habe mich so oft verlesen, daher kamen die ganzen doofen Fragen. Aber jetzt ist alles klar, dafür danke ich dir.


Zur Aufgabe gehört noch ein Teil c) dazu, aber ich habe das nicht gepostet, weil ich dachte, dass mit der restliche Beweis klar ist.


Aber dazu habe ich noch eine kleine Frage.




Teil c)

Das Minimalpolynom [mm] $\mu_{\alpha}$ [/mm] ist irreduzibel und [mm] $\Phi_{\alpha}$ [/mm] (also der Einsetzhomomorphismus)  induziert einen Isomorphismus

[mm] $K[\; [/mm] t [mm] \;] [/mm] / [mm] \langle \mu_{\alpha} \rangle \cong K[\; \alpha \; [/mm] ] = [mm] K(\alpha)$. [/mm]



Beweis
_______


Wäre [mm] $\mu_{\alpha}$ [/mm] nicht irreduzibel, so gäbe es eine Faktorisierung [mm] $\mu_{\alpha} [/mm] = f [mm] \cdot [/mm] g$ von [mm] $\mu_{\alpha}$ [/mm] in zwei Nicht - Einheiten und $f$ und $g$ hätten beide einen Grad, der echt kleiner als der Grad von [mm] $\mu_{\alpha}$ [/mm] wäre.


Zu gleich müsste $0 = [mm] \mu_{\alpha} (\alpha) [/mm] = (f [mm] \cdot g)(\alpha) [/mm] = [mm] f(\alpha) \cdot g(\alpha) [/mm] $

aber [mm] $\alpha$ [/mm] eine Nullstelle von $f$ oder $g$ sein, im Widerspruch zur Minimalität des Grades von [mm] $\mu_{\alpha}$ [/mm] in Teil a).

Also ist [mm] $\mu_{\alpha}$ [/mm] irreduzibel.


Daraus folgt, dass [mm] $K[\; [/mm] t [mm] \; [/mm] ] / [mm] \langle \mu_{\alpha} \rangle$ [/mm] ein Körper ist.

Wegen des Homomorphiesatzes gilt:


[mm] $K[\; [/mm] t [mm] \; [/mm] ] / [mm] \langle \mu_{\alpha} \rangle [/mm] = [mm] K[\; [/mm] t [mm] \; [/mm] ] / [mm] Ker(\Phi_{\alpha}) \cong Im(\Phi_{\alpha}) [/mm] = [mm] K[\; \alpha\; [/mm] ] [mm] \subseteq K(\alpha)$, [/mm]


so dass [mm] $K[\; \alpha \; [/mm] ]$ ein Zwischenkörper von $L/K$ ist, der [mm] $\alpha$ [/mm] enthält, womit auch [mm] $K(\alpha) \subseteq K[\; \alpha\; [/mm] ]$


Damit ist auch Teil c) gezeigt.





Meine letzte Frage:




Mit [mm] $K[\alpha]$ [/mm] bezeichnen wir das Bild des Einsetzhomomorphismus.

Also [mm] $K[\; \alpha \; [/mm] ] := [mm] Im(\Phi_{\alpha}) [/mm] = [mm] \{ f(\alpha) \; \vert \; f \in K[\; t \; ] \} \subseteq K(\alpha)$ [/mm]


Dass [mm] $K[\; \alpha \; [/mm] ] [mm] \subseteq K(\alpha)$ [/mm] habe ich mir klar gemacht.


Was ich nicht verstehe ist, warum [mm] $K[\; \alpha \; [/mm] ]$ ein Zwischenkörper  von $L/K$ ist und warum dann [mm] $K(\alpha) \subseteq K[\; \alpha \; [/mm] ]$ gilt.


Viele Grüße,

Tim

Bezug
                                        
Bezug
Existenz des Minimalpolynoms: Antwort
Status: (Antwort) fertig Status 
Datum: 18:33 So 05.04.2020
Autor: statler


> Meine letzte Frage:
>
> Mit [mm]K[\alpha][/mm] bezeichnen wir das Bild des
> Einsetzhomomorphismus.
>
> Also [mm]K[\; \alpha \; ] := Im(\Phi_{\alpha}) = \{ f(\alpha) \; \vert \; f \in K[\; t \; ] \} \subseteq K(\alpha)[/mm]
>  
>
> Dass [mm]K[\; \alpha \; ] \subseteq K(\alpha)[/mm] habe ich mir klar
> gemacht.
>  
>

Hallo Tim!

> Was ich nicht verstehe ist, warum [mm]K[\; \alpha \; ][/mm] ein
> Zwischenkörper  von [mm]L/K[/mm] ist und warum dann [mm]K(\alpha) \subseteq K[\; \alpha \; ][/mm]
> gilt.

Du hast dir doch klargemacht, daß [mm] $K[\alpha]$ [/mm] ein Körper ist. Er enthält offenbar $K$ und [mm] $\alpha$. $K(\alpha)$ [/mm] ist der 'kleinste' (in der mengentheoretischen Ordnung) Körper, für den das gilt. Deswegen ist [mm] $K(\alpha) \subseteq K[\alpha]$. [/mm]

Gruß aus der sozialen Distanz
Dieter

Bezug
Ansicht: [ geschachtelt ] | ^ Forum "Algebra"  | ^^ Alle Foren  | ^ Forenbaum  | Materialien


^ Seitenanfang ^
www.matheforum.net
[ Startseite | Forum | Wissen | Kurse | Mitglieder | Team | Impressum ]